Mobile Robot Control 2024 Ultron:Solution 2

From Control Systems Technology Group
Revision as of 12:12, 30 April 2024 by N.zhu@student.tue.nl (talk | contribs) (solution for exercise 2)
(diff) ← Older revision | Latest revision (diff) | Newer revision → (diff)
Jump to navigation Jump to search

Exercise2

Hao

  1. In map1 the robot can stop as the designed purpose.
    Exercise2-1 Hao.png
  2. In map2 the robot stopped when detected the wall on the right side with distance<=0.2
    Exercise2-2 Hao.png